You are on page 1of 42

PROJECT MANAGEMENT QUESTIONS AND ANSWERS 2024

1. An organization is under a business transformation. The project manager has been


assigned to a project related to one of the new business lines. What should the project
manager and team do before defining the scope of the project?
A. Lead a procurement meeting.
B. Create a work breakdown structure (WBS).
C. Adapt and tailor existing assets.
D. Sign off on the business case.
Solution: C. Adapt and tailor existing assets.

The project team will need to tailor existing assets to meet the project’s needs. The team
should take advantage of the organization’s existing assets, when appropriate, rather than
procuring new ones. The most effective strategy, in general, is to use existing assets that are
already in use within the organization. Procuring new assets isn't the best strategy and building
a WBS and signing off come after.
This question and rationale were developed in reference to:
https://www.pmi.org/disciplined-agile/process/asset-management/asset-management-
practices

2. A project team is arranging user testing to be completed for a project but does not have
a strong enough server to do so. The team orders a new server and postpones user testing.
What should the project manager do moving forward?

A. Eliminate the user testing from the project plan.


B. Utilize the current server to complete user testing.
C. Request to expedite the new server from the supplier.
D. Await the arrival of the new server and reevaluate timelines for user testing.
Solution: D. Await the arrival of the new server and reevaluate timelines for user testing.

The current server does not meet performance requirements for user acceptance testing. The
project team has no choice but to wait for the new server capable of supporting the testing and
then reevaluate timelines for user testing.
The other answer choices are incorrect. User testing is essential and cannot be eliminated;
using the improper infrastructure for testing can lead to invalid test results; and the project
manager may request the server be expedited, but that is not controllable and usually not
feasible due to logistics arrangement.
This question and rationale were developed in reference to:
Effective Project Management: Traditional, Agile, Extreme, Hybrid (No Date) Robert K.
Wysocki//6/ [Item]
| PMBOK Guide Sixth Edition (2018) PMI/PMI/9.3.1/330 [Item]
3. A research and development department is planning to develop a product that will
introduce a new line of business for the organization. What should the project manager do to
increase the project's chances of success?

A. Start developing the project management plan based on a previous project template
from the project management office (PMO).
B. Conduct an impact analysis of the new initiative to determine how the project should be
rolled out.
C. Plan a working session focusing on the scope, vision, and mission of the initiative.
D. Conduct benchmarking to determine the business viability of the initiative.
Solution: C. Plan a working session focusing on the scope, vision, and mission of the initiative

For any project success its critical to initially define vision and mission of the overall initiative
even before focusing on detailed planning.
Project management plan, determining business viability or impact analysis should be carried
out once the initiative is defined properly
This question and rationale were developed in reference to:
O'Reilly Platform (No Date) //Agile for Project Managers, Chapter 2 - Agile concepts/ [Item]
| PMBOK Guide Sixth Edition (2018) PMI/PMI/Project Resource Management/350 [Item]
4. While preparing a project's third progress report, the project manager identifies that the
project will experience delays due to late material deliveries. The two previous project reports
indicated that the project was on track. What should the project manager do next?

A. Discuss it with the project team to determine the most appropriate way to respond.
B. Add it to the issue log and include it in the project report.
C. Contact the procurement manager to expedite delivery of the material.
D. Escalate the issue to the project owner and request a schedule change.
Solution: B. Add it to the issue log and include it in the project report

PMBoK Sixth Edition in chapter 4 states,: "4.3.3.3 ISSUE LOG


Throughout the life cycle of a project, the project manager will normally face problems, gaps,
inconsistencies, or conflicts that occur unexpectedly and that require some action so they do
not impact the project performance. The issue log is a project document where all the issues
are recorded and tracked. Data on issues may include:
Issue type, Who raised the issue and when, Description, Priority, Who is assigned to the issue,
Target resolution date, Status, and Final solution.
The issue log will help the project manager effectively track and manage issues, ensuring that
they are investigated and resolved. The issue log is created for the first time as an output of this
process, although issues may happen at any time during the project. The issue log is updated as
a result of the monitoring and control activities throughout the project's life cycle"
The question identifies that there can be delays in the project schedule as planned due to
delivery issues. Therefore, the correct procedure for the project manager is to record the issue
in the Issue Log.
The Distractors are correctly written and are valid distractions
This question and rationale were developed in reference to:
Fundamentals of Technology PM, 2nd Ed (2nd) Colleen Garton, Erika McCulloch/Mc Press/9 -
Project Development/331 [Item]
| PMBoK, Sixth edition (2021) PMI/PMI/4/ [Item Issue Log]
| PMBOK® Guide--Fifth Edition (5th) PMI/PMI/10 - Project Communications Management/301
[Item]

5. After 20% of a project was completed, the customer made the decision to change the
provider for data center implementation. The new provider project manager requested a
meeting with the customer to clarify how the project will be impacted if the requirements are
not well gathered. Which are the project areas that could be impacted if the requirements are
not well gathered? (Choose 3.)
A. Cost
B. Quality assurance
C. Schedule
D. Quality customer service
E. Quality planning

Solution: A, C and E. Cost, Schedule and Quality planning

Data gathering techniques that can be used but are not limited:
Brainstorming
Interviews
Focus groups
Questionnaires and surveys
Benchmarking
This question and rationale were developed in reference to:
https://snippets.pmi.org/snippet/use-process-assets-and-define-scope/ [Item]

6. Since the beginning of a project, the product owner keeps asking during ceremonies
about the budget spent for each product iteration. The product owner seems much more
interested in the cost rather than the product itself. What should the project manager have
done earlier to change the product owner's behavior?

A. Worked on different approaches of estimating to give confidence of the cost spent in


each product increment
B. Worked on a communications management plan with reports of budget spent in each
iteration versus planned to avoid these discussions during ceremonies
C. Worked with the product owner to clarify their role in an agile project and the scope of
the agile ceremonies
D. Worked on a fixed-price contract to switch the product owner's attention to value
instead of money
Solution: C. Worked with the product owner to clarify their role in an agile project and the
scope of the agile ceremonies

The Project Manager should take this opportunity to mentor the Product owner on the role of
the team[Product Owner being a part of the team] in Agile projects, which appears to have
been missed earlier. Hence this is the correct answer.
The team is not concerned with the budgetary aspects in Agile projects. Hence the remaining
options are incorrect.
This question and rationale were developed in reference to:
O'Reilly Platform (No Date) //Essential Scrum - chapter 20 - Sprint execution/ [Item]
| The Agile Practice Guide (No Date) PMI/PMI/4/37 [Item Educate stakeholders around why
and how to be agile. Explain the benefits of business value based on prioritization, greater
accountability and productivity of empowered teams, and improved quality from more
frequent reviews, etc.]
7. A project manager is developing a project management plan to submit to the project
sponsor in two weeks. The project sponsor, however, wants to review the early resource
estimates before the deadline. What should the project manager do?

A. Ask the sponsor to wait until the Work breakdown structure (WBS) is done.
B. Share a draft copy of the Budget Estimate Sheet (BES) with the sponsor.
C. Re-share the Statement of Work (SOW) with the sponsor.
D. Share a draft copy of the Resource Breakdown Structure (RBS) with the sponsor.
Solution: D. Share a draft copy of the Resource Breakdown Structure (RBS) with the sponsor.

The RBS is a document that outlines the project's resources, including the types of resources,
the quantities of resources, and the costs of resources. The RBS visualizes how resources are
allocated across employees and teams at every stage of the project. This information would be
more relevant to the sponsor's request than the BES, WBS, and WBS dictionary for reviewing
the early resource estimates.
The other answer choices are incorrect. WBS Dictionary is a document that provides detailed
deliverable, activity, and scheduling information about each component in the work breakdown
structure. Work Breakdown Structure (WBS) is a hierarchical decomposition of the total scope
of work to be carried out by the project team to accomplish the project objectives and create
the required deliverables. Statement of work (SOW) is the all-encompassing scope to be carried
out by the project team to accomplish the project objectives.
This question and rationale were developed in reference to:
https://standardsplus.pmi.org/posts/Res_14/83b71ea9-307a-4362-9704-19e840cd65f1 [Item]
| Project Management Body of Knowledge (11/02/16) Project Management Institute//9.2.3.3/
[Item The resource breakdown structure is a representation of resources by category and type.
Examples of resource categories include but are not limited to labor, material, equipment, and
supplies. Resource types may include the skill level, grade level, required certifications, or other
information as appropriate to the project.]
8. A project manager sends out requests for regular status meetings and daily standup
meetings during the project setup phase. However, business stakeholders are declining the
meetings. What should the project manager do?

A. Check why meetings are declined


B. Re-share communications plan
C. Set up a project team ground rules
D. Hold a project kick-off meeting
Solution: A. Check why meetings are declined

The project manager should first check why the business stakeholders are declining the
meetings. There could be a number of reasons, such as they are not aware of the importance of
the meetings; they are too busy with other work; or they do not see the value in the meetings.
Once the project manager understands the reasons why the meetings are being declined, they
can take steps to address the concerns and ensure that the stakeholders are engaged and that
the meetings are productive.
The other options are incorrect. Re-sharing the communications plan, is not necessary if the
stakeholders have already received the plan.; setting up project team ground rules, is not
relevant to the situation as the stakeholders,not team members, are declining meetings; and
holding a project kick-off meeting, is too early in the project as the project setup phase has not
yet been completed.
This question and rationale were developed in reference to:
Agile Approaches on Large Projects in Large Organizations (No Date) Brian Hobbs and Yvan
Petit//2/2.7.4 [Item]
PMBOK Guide Seventh Edition (2022) ///[4.6.3 PLANS]
[2.4.4 COMMUNICATION]

9. A company outsources the execution of a project with a fixed-price contract to a


supplier. During execution, the supplier agrees to deliver additional customer requirements
that were not included in the contract. To fulfill the expanded scope, both parties add more
resources to keep the project on the planned schedule. Which technique is being applied in this
project?

A. Fast tracking
B. Crashing
C. Resource leveling
D. Lead and lag
Solution: B. Crashing

If the scope increases without increasing the schedule, the crashing technique achieves
compression by adding more resources.
This question and rationale were developed in reference to:
PMP: Project Management Professional Study Guide, Nineth Edition, 9th edition (2018) Kim
Heldman/Sybex/4/215 [Item]
| Project Management Body of Knowledge (11/02/16) Project Management Institute//6.5.2.6/
[Item Crashing: a technique used to shorten the schedule duration for the least incremental
cost by adding resources. Examples of crashing include approving overtime, bringing in
additional resources or paying to expedite delivery to activities on the critical path.]
10. A customer requested a change to a functionality that is being delivered in the next
iteration. The specialist responsible for this functionality was recently assigned to another
project. What should the project manager do first?

A. Request a new resource from HR.


B. Request a new resource from the project sponsor.
C. Negotiate for the resource's availability.
D. Tell the customer that the new change is not possible.
Solution: C. Negotiate for the resource's availability

The project manager should negotiate for the resource's availability first. The project manager
should explain to the project sponsor the impact of the customer's request on the project and
the need to have the specialist available to make the change. The project sponsor may be able
to negotiate with the other project manager to have the specialist temporarily assigned to the
current project.
The other answers are incorrect. If the project sponsor is unable to get the specialist's
availability, then the project manager can request a new resource from HR. The project
manager should not tell the customer that the new change is not possible without first trying to
negotiate for the resource's availability or requesting a new resource from HR.
This question and rationale were developed in reference to:
https://standardsplus.pmi.org/posts/res_id17_org_article_fix1/3210b0c2-3297-4312-9729-
56382e737be3 [Item]
| PMI.org (2006) //https://www.pmi.org/learning/library/matrix-evolves-internal-
conflicts4043// [Item]

11. What can a project manager do to communicate the formal project announcement and
relevant information to stakeholders, and then gain their commitment?

A. Create the project charter.


B. Conduct a kick-off meeting.
C. Develop the communications management plan.
D. Prepare and distribute the responsible, accountable, consult, and inform (RACI) matrix.
12. After completing the stakeholder analysis, the project manager is in the action planning
phase. What should the project manager do with the stakeholders that have no interest or
influence in the project?

A. Closely monitor these stakeholdersSolution: C. Developed communications plan.


B.
C. By developing a communications plan, the project manager can ensure that the
stakeholders are kept informed of the project's progress and that they have a way to
communicate their concerns. This can help to avoid the situation where stakeholders are
complaining and declining meetings.
D.
E. The other options are incorrect. Creating the project charter is a good idea, but it is not
the most important thing to do in this situation. The project manager needs to address the
issue of stakeholder communication first. Conducting a kick-off meeting is a good way to
communicate with stakeholders, but it is not enough on its own. The project manager needs to
have a plan for ongoing communication with stakeholders. Distributing a RACI matrix is a tool
that can be used to clarify the roles and responsibilities of the stakeholders. However, it is not a
substitute for a comprehensive communications plan.
F. This question and rationale were developed in reference to:
G. e-Reads (No Date) //Effective Project Management: Traditional, Agile, Extreme, Fifth
Edition./Chapter 5 - Conducting the Project Kick‐Off Meeting [Item]
H. | Fundamentals of Technology PM, 2nd Ed (2nd) Colleen Garton, Erika McCulloch/Mc
Press/6/195 [Item]
I. | PMBoK, Sixth edition (2021) PMI/PMI/3/ [Item Project Initiation and Project
Communications Management]
J.
K. Highly encourage these stakeholders
L. Attentively focus on these stakeholders
M. Visibly harness these stakeholders
Solution: A. Closely monitor these stakeholders

Stakeholders who have no interest or influence in the project may not be actively involved in
the project, but they may still be affected by the project's outcome. It is important to keep
these stakeholders informed of the project's progress and address any concerns. By closely
monitoring these stakeholders, the project manager can ensure that they are not negatively
impacted by the project and that they are aware of the project's progress.
The other answer choices are incorrect. Highly encouraging these stakeholders is not necessary
because they are not interested in the project, so they are unlikely to be persuaded to become
more involved. Attentively focusing on these stakeholders is not necessary because they are
not influential, so they do not need to be given a lot of attention. Visibly harnessing these
stakeholders is not necessary. These stakeholders are not interested in the project, so they are
unlikely to be willing to help the project succeed.
This question and rationale were developed in reference to:
https://www.pmi.org/learning/library/engaging-stakeholders-project-success-11199

13. The team and product owner are reviewing user stories in the iteration planning
meeting. While reviewing stories, it is identified that some of the high-value stories have more
associated risks. What should the agile project manager do?

A. Recommend that the team include risky stories in early iterations rather than later ones.
B. Recommend that the team choose stories with low risk for easy completion to keep
team motivation high.
C. Work with the product owner to decide on the stories that should go during the
iteration.
D. Use the Pareto analysis to decide which stories can be completed the fastest.
Solution: A. Recommend that the team include risky stories in early iterations rather than later
ones

Risk management should be iterative in agile projects. Any risk items or stories that are
identified as higher risks should be addressed earlier by adding them in appropriate sprints
rather than to wait for the risks to hit the project.
The other answer choices are incorrect. Focusing on low risk stories will leave out the high risk
stories that should be prioritized; the team, not the product owner, should determine the
stories; Pareto analysis is not the right tool to use.
This question and rationale were developed in reference to:
O'Reilly Platform (No Date) //10 Agile Risk Management/ [Item Agile for Project Managers : ]
| O'Reilly Platform (No Date) //7/ [Item Lean-Agile Software Development, Guy Beaver]
14. A project manager is leading a project with seven key stakeholders. What should the
project manager consider as the most important part of stakeholder management?

A. Avoid challenging stakeholders.


B. Make changes to the project schedule in real-time.
C. Ensure all stakeholders receive the same information.
D. Establish good working relationships with all stakeholders.
Solution: D. Establish good working relationships with all stakeholders.

Relationships between the project manager, the project’s stakeholders, and the project team
are very important. The most critical part of stakeholder management is relationships.
This question and rationale were developed in reference to:
https://www.pmi.org/learning/library/project-relationships-stakeholder-circle-8092 [Item
Project relationships are those relationships that occur between the project manager and the
project’s stakeholders as well as those that occur among the project stakeholders themselves.
This network, comprising all the relationships both within and around the project ( Bourne &
Walker, 2003; Briner, Hastings & Geddes, 1996; Frooman, 1999), forms the project
environment, or sphere of influence and support, on which a project depends for its very
existence and has to be managed.]
15. A project manager has just started building the team for a huge and complex project.
The project manager is highly interested in having effective communication with their large
team. What should the manager do to achieve this?

A. Organize team activities to help team members express their interests.


B. Establish a process of sharing information with all team members.
C. Use timely and clear assessment of the situation with team members.
D. Allow free exchange of ideas and opinions between team members.
Solution: B. Establish a process of sharing information with all team members.

Building relationships with your stakeholders also helps to build trust between you, them, and
your project team – and trust around the project. Trust is a key component of effective
communication with others. Stakeholders who feel they can trust you are more apt to share
information with you and be engaged in the project.
The other options are incorrect. Organizing team activities to help team members express their
interests, might help to build morale and teamwork, but it would not necessarily improve
communication. Using timely and clear assessment of the situation with team members is
important, but it is not enough on its own. The project manager also needs to have a process
for sharing information with team members. Allowing free exchange of ideas and opinions
between team members is important, but it is not enough on its own. The project manager also
needs to have a process for ensuring that all team members are aware of the project's progress
and that they have a way to communicate their ideas and concerns.
This question and rationale were developed in reference to:
https://www.pmi.org/learning/library/managing-communications-effectively-efficiently-5916
[Item Arredondo, 2000). ]
16. A project team often asks questions about the rationale for their work and how the
client perceives their progress and deliverables. What should the project manager do to
build trust and get the team engaged?
A. Share feedback and invite team members to status meetings with the stakeholders.
B. Assure the team that the client is happy and you are handling the expectations.
C. Present the project plan and progress report to the team with regular updates when
revised
D. Compare the pending tasks to the completed tasks so the team has a sense of
accomplishment.
Solution: A. Share feedback and invite team members to status meetings with the stakeholders
The concern is based on the relationship with the client so we should engage the team
members on that process of the project.
This question and rationale were developed in reference to:
Effective PM: Trad, Agile, Extreme (2015) Robert K. Wysocki/Wiley/Chapter 10: Agile Project
Management/Implementing APM Projects [Item Adding more functions and features to the
solution and implementing them at the same time sounds great. The client and the end user
can benefit from whatever business value can be attained, experience the solution unfolding
over short time periods, work with the solution, and provide valuable feedback to the
developers about further additions and changes to the solution. But there is another side to this
story, and that is the implementation of a constantly evolving solution. Iterations and cycles are
short duration—2 to 4 weeks is typical. The end users will give up and surrender if you expect
them to change how they do their work by implementing a new solution every few weeks. How
about your organization? What is its organizational velocity? Can it absorb change that fast?
Most can't or won't. So what are the client and the project manager to do? Getting frequent
client feedback is critical to discovering the complete solution and ultimately to project success,
but the organization can't absorb change as fast as the APM models would like. There is also
the question of the project team's ability to support frequent releases. Training,
documentation, and a support group are needed. Let's see, what release are you using
again?https://learning.oreilly.com/library/view/effective-project-
management/9781118729311/9781118729311c10.xhtml]
| https://snippets.pmi.org/snippet/7-ways-to-build-your-leadership-skills/ [Item]
17. A supplier for a regulatory project is responsible for making the component changes. The
supplier is unlikely to meet the project schedule. Based on the risk response plan, the project
manager secured stakeholder approval to use paper forms until the component changes go
live. It is still possible, however, that the paper form changes may not be ready in time.
Which risk type is the project manager facing in this situation?
A. Secondary risk
B. Residual risk
C. Primary risk
D. Compliance risk
Solution: B. Residual risk
The correct answer is a residual risk.
Residual risks are the leftover risks that are expected to remain after the planned response of
risk has been taken. In this case, the residual risk is the chance that the paper form changes
may not be ready in time; there isn't much the project manager can do to prevent this delay.
A residual risk is a risk that remains after risk responses have been implemented.
A secondary risk is a risk that arises as a direct result of implementing a risk response.
Risk responses, when implemented, can have potential effects on the objectives and as such,
can generate additional risks. These are known as secondary risks and are analyzed and planned
for in the same way as those risks that were initially identified. There may be residual risks that
remain after the responses are implemented. The risk is that the supplier is unlikely to meet the
project schedule. Even after consulting the risk response plan, it is still possible that the risk
remains.
This question and rationale were developed in reference to:
https://www.pmi.org/learning/library/overall-project-risk-assessment-models-1386 [Item]
| PMI.org (2016) //https://www.pmi.org/learning/library/isd-project-performance-residual-
risk-10223// [Item]
The Standard for Risk Management in Portfolios, Programs, and Projects (2019)
Solution: C. Integrate skills development activities.
When a project manager needs to assign new resources to an existing project team, it is
important to integrate skills development activities into the project execution phase. This will
help to ensure that all team members have the skills and knowledge they need to do their jobs
effectively and meet project requirements on time. A highly skilled and capable team is more
likely to work effectively and meet project requirements on time. Skills development activities
ensure that team members are equipped to handle their roles and responsibilities competently.
The other options are incorrect because they are not the most immediate actions to ensure the
team's effectiveness during the execution phase of the project.
Creating accessible library services is incorrect because it does not directly address the team's
skills and capabilities and does not ensure effectiveness in the execution phase.
Conducting brainstorming sessions is incorrect because they are not directly related to skills
development or ensuring the team's effectiveness.
Monitoring team cohesiveness is important for assessing team dynamics and collaboration, but
it is not a proactive way to ensure the team works effectively and meets project requirements
on time. The project manager should focus on helping the new resources integrate into the
team and develop the skills they need.
This question and rationale were developed in reference to:
The AMA Handbook of Project Management, Third Edition (No Date) /AMACOM/12/Fast Track
to Teaming [Item]
PMBOK Guide Seventh Edition (2022) /// [2.4.3 PROJECT TEAM COMPOSITION AND
STRUCTURE] [2.4.1 INTERNAL ENVIRONMENT]

Solution: C. Risk Register, Assumption Log, Issue Log, Risk Report


The correct answer is risk register, assumption log, issue log, and risk report.
At a minimum, the risk register, issue log, assumption log, and risk report should be reviewed
for revision upon completion of the qualitative risk analysis. Other documents, such as the
lessons learned database and stakeholder register may require an update, but it is likely that
the four above will be affected. The other answers are either incomplete or include possible
optional but not the most likely affected documents.
The next step after qualitative risk analysis is to update those all-important project documents.
These include:
Risk register: The results of qualitative risk analysis will largely be added to the risk register.
These items include probability and impact assessments for each individual project risk, as well
as priority levels, identified risk owners, risk urgency information, and risk categories. Risks
judged to be a high priority should be separated for further analysis and response planning (in
general, they should also be monitored more frequently). A watch list should also be created
for low-priority risks.
Risk report: The risk report should be updated to indicate the overall project risk and the most
important individual project risks, sorted by category and trends. It should also include a
prioritized list and summary of all identified risks.
Assumption log: The log should be updated if new assumptions have come to light or new
constraints have been identified during the process.
Issue log: This should be updated to capture any new issues you’ve discovered or changes in
currently logged issues.
This question and rationale were developed in reference to:
https://standardsplus.pmi.org/posts/idbeta5/13aec16b-9482-47f7-88e6-72e3449b604f

19. A vendor informs a project manager that the project's equipment can be delivered earlier
than anticipated. The project manager recognizes that this is an opportunity to complete the
project ahead of schedule.
What should the project manager do next?
A. Request management approval.
B. Use the equipment offered.
C. Revise the project schedule.
D. Determine if there is a risk.
Solution: D. Determine if there is a risk
The project manager should determine if there is a risk before making any decisions. The early
delivery of equipment could create a risk if the project team is not prepared to use it or if the
vendor is unable to meet the quality standards. The project manager should assess the impact
of the early delivery on the project schedule and budget, and review the vendor's quality
assurance process to ensure that the equipment meets the project's requirements.
The other answer choices are incorrect. Requesting management approval is important, but it
should not be done before the project manager has determined if there is a risk. Using the
equipment offered could create a risk if the project team is not prepared to use it or if the
vendor is unable to meet the quality standards. Revising the project schedule should not be
done before the project manager has determined if there is a risk.
This question and rationale were developed in reference to:
PMBOK Guide Sixth Edition (2018) PMI/PMI// [Item]
| PMBoK, Sixth edition (2021) PMI/PMI/3, 9 and 11/ [Item Identify Risks
Acquire Resources]

20. The project manager is having difficulties persuading stakeholders to adopt new technlogy
in the project.
What should the project manager have done to avoid this resistance?
A. Invest in stakeholders' communication
B. Provide technical support to stakeholders
C. Propose an agreement to stakeholders
D. Engage stakeholders as early as possible
Solution: D. Engage stakeholders as early as possible
The best way to avoid resistance to new technology is to engage stakeholders as early as
possible. This means involving them in the decision-making process from the beginning and
giving them a chance to provide input on the new technology. By engaging stakeholders early
on, the project manager can assess their needs and concerns, address objections, build trust
and rapport, and get their buy-in.
The other answer choices are incorrect. Investing in stakeholders' communication is important,
but it is not enough to avoid resistance. The project manager also needs to engage stakeholders
early on. Providing technical support to stakeholders may help to address some of their
concerns, but it is not enough to overcome their resistance. Proposing an agreement with
stakeholders may be a good way to get their buy-in, but it is not as effective as engaging them
early on.
This question and rationale were developed in reference to:
https://www.pmi.org/learning/library/improve-stakeholder-management-9901 [Item Utilizing
change management tools and techniques can help a project or program manager more
effectively identify, plan, and engage stakeholders. This targeted approach can strengthen
stakeholder assessment capabilities to build a better stakeholder management plan,
communications plan, schedule, budget, and other project management tools. It can also help
build value into the delivery of the changes themselves by strengthening adoption]
21. A project incurs a 4-week delay. If delayed another 2 weeks or longer, the project will be
penalized. In order to be able to deliver on time, the team is offered an appealing financial
compensation and 2 days of vacation.
What theory is being applied?
A. Achievement theory
B. Expectancy theory
C. Maslow’s theory
D. Contingency theory
Solution: B. Expectancy theory
Expectancy theory proposes that employees are motivated when they are given assignments
they feel confident they can achieve, when they value the compensation you offer and when
they believe you will compensate them as promised.
The aim of need achievement theory is to explain why certain individuals are more motivated
to achieve than others.
Maslow's hierarchy of needs is a theory of motivation which states that five categories of
human needs dictate an individual's behavior. Those needs are physiological needs, safety
needs, love and belonging needs, esteem needs, and self-actualization needs.
The contingency theory emphasizes the importance of both the leader's personality and the
situation in which that leader operates.
This question and rationale were developed in reference to:
https://www.pmi.org/learning/library/motivating-construction-productivity-learning-
disciplines-5731 [Item Expectancy theory argues that the motivational force to perform or
expend effort is a multiplicative function of the expectancies that individuals have concerning
future outcomes and the value they place on those outcomes.]
| PMP: Project Management Professional Study Guide, Nineth Edition, 9th edition (2018) Kim
Heldman/Sybex/9/408 [Item]
22. A supplier reports that they are over capacity, which may delay their delivery. What should
the project manager update?
A. Risk register
B. Stakeholder register
C. Issue log
D. Project schedule
Solution: A. Risk register
PMBoK Sixth Edition Chapter 4 and 11 states: "
11.2.3.1 RISK REGISTER
The risk register captures details of identified individual project risks. The results of Perform
Qualitative Risk Analysis, Plan Risk Responses, Implement Risk Responses, and Monitor Risks
are recorded in the risk register as those processes are conducted throughout the project. The
risk register may contain limited or extensive risk information depending on project variables
such as size and complexity.
On completion of the Identify Risks process, the content of the risk register may include but is
not limited to:
List of identified risks. Each individual project risk is given a unique identifier in the risk register.
Identified risks are described in as much detail as required to ensure unambiguous
understanding. A structured risk statement may be used to distinguish risks from their cause(s)
and their effect(s).
Potential risk owners. Where a potential risk owner has been identified during the Identify Risks
process, the risk owner is recorded in the risk register. This will be confirmed during the
Perform Qualitative Risk Analysis process.
List of potential risk responses. Where a potential risk response has been identified during the
Identify Risks process, it is recorded in the risk register. This will be confirmed during the Plan
Risk Responses process.
Additional data may be recorded for each identified risk, depending on the risk register format
specified in the risk management plan. This may include: a short risk title, risk category, current
risk status, one or more causes, one or more effects on objectives, risk triggers (events or
conditions that indicate that a risk is about to occur), WBS reference of affected activities, and
timing information (when was the risk identified, when might the risk occur, when might it no
longer be relevant, and what is the deadline for taking action).
4.4.3.1 LESSONS LEARNED REGISTER
The lessons learned register can include the category and description of the situation. The
lessons learned register may also include the impact, recommendations, and proposed actions
associated with the situation. The lessons learned register may record challenges, problems,
realized risks and opportunities, or other content as appropriate.
The lessons learned register is created as an output of this process early in the project.
Thereafter it is used as an input and updated as an output in many processes throughout the
project. The persons or teams involved in the work are also involved in capturing the lessons
learned. Knowledge can be documented using videos, pictures, audios, or other suitable means
that ensure the efficiency of the lessons captured.
At the end of a project or phase, the information is transferred to an organizational process
asset called a lessons learned repository"
Therefore, the correct answer is: "Risk Register", because that is where all the risks related to
the project should be recorded.
The Distractors are correctly worded and accepted
This question and rationale were developed in reference to:
PMBoK, Sixth edition (2021) PMI/PMI/4 and 11/ [Item Risk Register and Lessons Learned
Register]
| PMBOK Guide--Fifth Edition (5th) PMI/PMI/11.6/349 [Item]
®

| Project Risk Management Guidelines: Managing Risk in Large Projects and Complex
Procurements (2005) Dale Cooper, Stephen Grey, Geoff Raymond and Phil Walker/John Wiley &
Sons/7/ [Item]
23. Which is the process that the project manager performs to prioritize individual project risks
for further analysis or action by assessing their probability of occurrence and impact, as well as
other characteristics?
A. Quantitative Risk Analysis
B. Qualitative Risk Analysis
C. Identify Risks
D. Plan Risk Management
Solution: B. Qualitative Risk Analysis
The choices list four processes within Risk Management. They are done in the order of 1) Plan
Risk Management, 2) Identify Risks, 3) Qualitative Risk Analysis, 4) Quantitative Risk Analysis.
Qualitative Risk Analysis prioritizes identified risks by assessing their probability and impact,
and is relatively simple, compared to Quantitative Risk Analysis. Quantitative analysis uses
mathematical models to calculate risk responses, and can be time consuming and expensive.
Therefore, only the highest priority risks are advanced to quantitative analysis.
This question and rationale were developed in reference to:
https://standardsplus.pmi.org/posts/idbeta5/13aec16b-9482-47f7-88e6-72e3449b604f [Item
This process allows you to prioritize individual project risks for further analysis or action so you
can focus your energy on the risks that pose the greatest threat to the project.]
24. A project's outcome is highly sensitive to and dependent upon financial market activity.
Therefore, the company's financial advisor delivers daily exchange-rate forecasts. The project's
budget was planned using a conservative scenario with high exchange-rate variations. The
project manager now wants to review this budget to update the probability and impact of a
possible new scenario using current financial conditions.
To what should the project manager refer?
A. Stakeholder register
B. Risk register
C. Lessons learned
D. Marketplace conditions
Solution: B. Risk register
The correct answer is the risk register.
A risk register is a repository in which outputs of risk management processes are recorded. The
risk register captures details of identified individual project risks. Information in a risk register
can include the person responsible for managing the risk, probability, impact, risk score,
planned risk responses, and other information used to get a high-level understanding of
individual risks.
The other answers are incorrect because they do not contain all of the information needed.
A stakeholder register is incorrect because a stakeholder register is a project document that
includes information about project stakeholders including an assessment and classification of
project stakeholders.
Lessons learned is incorrect because they reflect the knowledge gained during a project, which
shows how project events were addressed or should be addressed in the future, for the
purpose of improving future performance.
Marketplace conditions are factors external to the organization that can enhance, constrain, or
have a neutral influence on project outcomes. Marketplace conditions include competitors,
market share, brand recognition, technology trends, and trademarks. The marketplace
conditions option is incorrect because the company's financial advisor delivers daily exchange-
rate forecasts, and the project manager wants to review the budget to update the probability
and impact of a possible new scenario.
This question and rationale were developed in reference to:
PMBOK Guide Seventh Edition (2022) /// [Item] | PMBoK, Sixth edition (2021) PMI/PMI/11/
[Item Risk Register]
25. Which of the following combination of approaches should a team lead take to have a highly
motivated and high-performing project team? (Choose three)
A. Build strong relationships, Provide context to work and the big picture, Implement
customized incentives.
B. Ensure team peer review is done, Emphasize on team performance management,
Prioritize individual rewards
C. Emphasize on team dynamics, Plan and deliver regular training on team dynamics, Hold
regular team building activities
A. Foster a competitive environment, Prioritize reward for team member excellence, Hold
regular team meetings
Solution: A, B and C. Build strong relationships, Provide context to work and the big picture,
Implement customized incentives, Ensure team peer review is done, Emphasize on team
performance management, Prioritize individual rewards and Emphasize on team dynamics, Plan
and deliver regular training on team dynamics, Hold regular team building activities
Competitive environments don't help with motivation and high performance for all team
members.
This question and rationale were developed in reference to:
https://snippets.pmi.org/snippet/motivate-teams-to-achieve-better-outcomes/ [Item]
26. Due to increased price volatility, a vendor asks the project manager of a large development
project to make a price change that will affect the project contract.
What should the project manager do?
A. Request the change control board (CCB) to meet with the vendor to discuss the change.
B. Execute the price change requested by the vendor and update the project contract.
C. Comply with the vendor's request and update the project management plan.
D. Analyze the impact and submit a formal change request, if appropriate.

Solution: D. Analyze the impact and submit a formal change request, if appropriate.
The project manager should thoroughly asses how the proposed price change will affect various
aspects of the project, including scope, budget, schedule, and quality. If it is determined that
the vendor's request would significantly impact the project, such as changes to the project's
cost, scope, or schedule, the project manager should submit a formal change request.
Submitting a formal change request is the appropriate way to document and communicate the
proposed change to relevant stakeholders and decision-makers.
Requesting a meeting with the CCB and the vendor is not the most efficient approach. It is
important to keep the CCB informed of any potential changes to the project, but the project
manager should first assess the impact of the price change before requesting a meeting.
Complying with the vendor's request and executing the price change without prior analysis and
approval disregards the need to understand how the change affects the overall project. The
price change may significantly impact the project's scope, budget, schedule, and other aspects,
potentially leading to uncontrolled project changes. Further, it is important to get input from all
stakeholders before making a decision.
This question and rationale were developed in reference to:
PMBOK Guide Seventh Edition (2022) PMI/PMI/ [2.4.7 CHANGES]
27. A project manager with a directive behavioral orientation is leading a virtual team of
software developers. The project manager wants to build a high-performing team that is
motivated and productive. The team is working on a complex project with a tight deadline.
How should the project manager lead this team?
A. Build rapport and develop strong relationships with team members.
B. Motivate team members with achievement awards.
C. Influence team decisions and control the delivery process.
D. Implement communication channels to announce task completion.
Solution: D. Implement communication channels to announce task completion.
In this situation, the project manager has a directive behavioral orientation and is task-oriented
and results-driven. Directive orientation is characterized by a focus on results, a willingness to
take charge, and a preference for clear and concise communication. These qualities are
essential for a project manager who is responsible for ensuring that a complex project with a
tight deadline is completed successfully.
The other answer choices are incorrect. Building rapport and relationships is aligned with
affiliation orientation, motivating team members is aligned with achievement orientation, and
influencing and controlling team members is aligned with power orientation.
This question and rationale were developed in reference to:
https://www.pmi.org/learning/library/successful-motivational-techniques-virtual-teams-8161
[Item]
28. A large risk to a project is identified, and the risk management plan is executed. Upon
completion of the approved actions, what should the project manager do?
A. Update the risk management plan.
B. Revise the project schedule and the budget.
C. Update the lessons learned.
D. Identify new risks, and monitor residual risks.
Solution: D. Identify new risks, and monitor residual risks
PMBoK Sixth Edition states: "11.2 IDENTIFY RISKS
Identify Risks is the process of identifying individual project risks as well as sources of overall
project risk, and documenting their characteristics. The key benefit of this process is the
documentation of existing individual project risks and the sources of overall project risk. It also
brings together information so the project team can respond appropriately to identified risks.
This process is performed throughout the project
Identify Risks considers both individual project risks and sources of overall project risk.
Participants in risk identification activities may include the following: project manager, project
team members, project risk specialist (if assigned), customers, subject matter experts from
outside the project team, end users, other project managers, operations managers,
stakeholders, and risk management experts within the organization. While these personnel are
often key participants for risk identification, all project stakeholders should be encouraged to
identify individual project risks. It is particularly important to involve the project team so they
can develop and maintain a sense of ownership and responsibility for identified individual
project risks, the level of overall project risk, and associated risk response actions"
The correct answer is: "Identify new risks and monitor residual risks"
The distractors are valid
This question and rationale were developed in reference to:
e-Reads (No Date) ///Modern Corporate Risk Management: A Blueprint for Positive Change and
Effectiveness by Glenn Koller, Chapter 7 - Solutions to the Problems-The Aftermath [Item]
| e-Reads (No Date) ///Practical Project Risk Management: The ATOM Methodology by David
Hillson and Peter Simon, Chapter 9 - Just Do It (Implementation) [Item]
| PMBoK, Sixth edition (2021) PMI/PMI/11/ [Item Identify risks]
| PMBOK Guide--Fifth Edition (5th) PMI/PMI//Chapter A1.7.9 control risk, pg 457 [Item]
®

29. A project manager is leading a project to develop a new system for a government agency
using a predictive approach. The project's governance framework is designed to ensure that the
system meets the needs of the stakeholders and complies with all applicable regulations. The
project manager receives an approved change request related to a project's governance
framework.
What should the project manager do to implement this change?
A. Review the changes to the governance framework and update relevant planning
documents.
B. Recalculate the durations of project activities affected by the change to the governance
framework.
C. Put the project on hold until the impact of the governance framework can be assessed.
D. Submit the formal change request to the project steering committee for final approval.
Solution: A. Review the changes to the governance framework and update relevant planning
documents.
The project manager needs to fully understand the nature and extent of the approved changes.
Without a thorough review, it's impossible to know how these changes will impact the project.
After reviewing the changes, the project manager should update planning documents to ensure
that the project aligns with the new governance framework and maintains alignment with
stakeholder needs and regulatory requirements.
Recalculating the durations of project activities is not necessary until the project manager has
reviewed the changes and determined their impact on the project.
Putting the project on hold is not necessary unless the changes require a major overhaul of the
project plan. Putting the project on hold could be costly and disruptive; the focus should be on
implementing the approved changes effectively.
Submitting the formal change request to the project steering committee is not necessary, as
the change request has already been approved.
This question and rationale were developed in reference to:
Adv PM: A Structured Approach 4th Ed (Jan 1 2004 12:00AM) Frederick Harrison and Dennis
Lock/Gower Publishing Co.//Change implementation and integration, page 253 [Item]
PMBOK Guide Seventh Edition (2022) /// [2.5.7 MONITORING NEW WORK AND CHANGES]
30. After weeks of dealing with conflicts, compromising, and achieving the project goal, team
members are focusing on documenting the lessons learned to be shared with stakeholders,
sponsors, and future project teams.
What phase are they most likely in?
A. Forming
B. Storming
C.Performing
D. Adjourning
Solution: D. Adjourning
During this final phase, the successful completion of the project is often celebrated, and the
team members focus on documenting the lessons learned as part of the project closure
activities. A team that has successfully built trusting relationships is likely to be better equipped
to honestly evaluate its processes, issues, and achievements to glean the lessons that can be
shared with stakeholders, sponsors, and future project teams.
The other answer choices are incorrect. They describe prior stages before lessons learned are
documented to be shared with stakeholders, sponsors, and future project teams. The team has
already handled conflicts, made compromises, and achieved the project goal, the team would
then enter the adjourning stage.
This question and rationale were developed in reference to:
Building a trust-based team (pmi.org)
PMBOK Guide Seventh Edition (2022) ///[4.2.6.1 Tuckman Ladder]
31. A company, which is conducting a critical bidding process, included a penalty clause in an
agreement specifying that the selected supplier must pay a significant liquidated damages
penalty in the event of non-performance. Given that this bidding process is extremely attractive
to the supplier with a medium risk threshold, how should the supplier's project manager
budget for it?
A.Increase the quote to mitigate the risk and support the above clause.
B.Advise management not to participate in the request for proposals (RFP) since it is a big risk.
C.Advise management to purchase insurance coverage to transfer the risk and include it in the
budget.
D.Record it as a risk in the risk register and review it regularly to avoid the penalty.
Solution: C. Advise management to purchase insurance coverage to transfer the risk and
include it in the budget.
Transferring a risk involves shifting ownership of a threat to a third party to manage the risk
and to bear the impact if the threat occurs. Risk transfer often involves payment of a risk
premium to the party taking on the threat. Transfer can be achieved by a range of actions
including, but not limited to, the use of insurance, performance bonds, warranties, guarantees,
etc. Agreements may be used to transfer ownership and liability for specified risks to another
party. Transferring the risk is a preferable strategy for average risk threshold takers.
Increasing the quote for this project may lead to not getting this project, especially when the
bidding process is attractive for the vendor/supplier with an average risk threshold.
Avoiding the risk solely based on penalty charges is incorrect because the project can't be
eliminated, especially when it is attractive for the vendor/supplier with an average risk
threshold.
Recording the penalty charge as a risk in the risk register alone is not going to help.
This question and rationale were developed in reference to:
PMBOK Guide Sixth Edition (2018) PMI/PMI/11 Project Risk Management/443
PMBOK Guide Seventh Edition (2022) ///[2.8.5.1 Threats] [2.8.5 RISK] [2.5.6 WORKING WITH
PROCUREMENTS]
The Standard for Risk Management in Portfolios, Programs, and Projects, 2019 [4.6 PLAN RISK
RESPONSES]
| PMI.org (1999) //https://www.pmi.org/learning/library/effective-project-risk-management-
process-5311// [Item]
32. The degree to which risk management is pursued can be the difference between success
and failure. In an attempt to avoid project failure, what risks should be assessed and
categorized?

A.All identified risks


B.Only risks identified as high probability of occurrence
C.Only risks identified as high potential impact
D.All risks identified as high probability of occurrence and high potential impact
Solution: A. All identified risks
The correct answer is all identified risks.
Risk management allows organizations and teams to increase the predictability of outcomes,
both qualitatively and quantitatively. The project manager should assess and categorize all
possible risks. Project team members should proactively identify risks throughout the project to
avoid or minimize the impacts of threats and trigger or maximize the impacts of opportunities.
Both threats and opportunities have a set of possible response strategies that can be planned
for implementation should the risk occur. The remaining options are incorrect because failing to
assess and categorize all risks leaves the team unprepared and vulnerable.
This question and rationale were developed in reference to:
https://www.projectmanagement.com/articles/421532/Effectively-Managing-Risks-During-
Project-Execution [Item]
PMBOK Guide Seventh Edition (2022) /// [Item]
33. After the kickoff meeting for an industrial project, the assigned project manager must lead
the team in identifying project risks. As team members are distributed in 6 countries, which risk
identification method is best used in this situation?
A. Affinity diagram
B .Decision tree analysis
C. The Delphi technique
D. Monte Carlo analysis
Solution: C. The Delphi technique
The correct answer is the Delphi technique.
The Delphi technique uses facilitated anonymous polling to identify risks. The facilitator gathers
the initial poll responses and circulates them without attribution to the entire group. The group
members may then revise their contributions based on those of others. The process often
generates a consensus after a few iterations. This makes it an appropriate method to use when
participants can’t meet in one place, like the team members in 6 different countries.
The other answers are incorrect because they are not risk identification techniques, they are
risk analysis techniques.
Monte Carlo analysis is a quantitative risk analysis technique that computes or iterates the
project cost or project schedule many times using input values, selected at random from
probability distributions of possible costs or durations, to calculate a distribution of possible
total project costs or completion dates.
Decision tree analysis is a quantitative risk analysis technique used to determine partial and
global probabilities of occurrence
An affinity diagram is a qualitative risk analysis technique used to organize specific ideas or
factors that contribute to a risk. It helps to sort risks by similarities or generic risk categories.
This question and rationale were developed in reference to:
Agile Practice Guide (2017) PMI/PMI// [Item]
| PMBOK Guide Sixth Edition (2018) /// [Item]
| Practice Standard for Project Risk Management (7/1/2009) Project Management
Institute/PMI// [Item]
34. Throughout the life cycle of a project, the project manager will normally face problems,
gaps, inconsistencies, or conflicts that occur unexpectedly. This requires some action so that
project performance is not impacted.
Which project document will be used to record and track this?
A. Issue log
B. Risk register
C. Lessons learned
D. Project management plan
Solution: A. Issue log
The correct answer is the issue log.
The issue log is a project document where all the issues are recorded and monitored. An issue is
a current condition or situation that may have an impact on the project objectives. An issue log
is used to record and monitor information on active issues. Issues are assigned to a responsible
party for follow up and resolution.
A risk register is a repository in which outputs of risk management processes are recorded.
The project management plan is the document that describes how the project will be executed,
monitored and controlled, and closed.
Lessons learned is the knowledge gained during a project, which shows how project events
were addressed or should be addressed in the future, for the purpose of improving future
performance.
This question and rationale were developed in reference to:
PMBOK Guide Sixth Edition (2018) PMI/PMI/3 The Role of Project Manager/96 [Item 4.3.3.3
ISSUE LOG]
35. A project manager is leading a complex initiative with diverse stakeholders and must
cultivate strong stakeholder relationships. The project manager also has to build a shared
understanding of the project with all stakeholders.
What should the project manager do?
A. Facilitate interactive workshops to discuss goals and expectations.
B. Conduct detailed stakeholder mapping exercises.
C. Deliver formal presentations summarizing project updates.
D. Distribute comprehensive project documentation for individual review.
Solution: A. Facilitate interactive workshops to discuss goals and expectations.
By facilitating interactive workshops, the project manager can cultivate strong stakeholder
relationships and build a shared understanding of the project.
The other answer choices are incorrect. While stakeholder mapping is crucial for identifying and
understanding stakeholder interests, it primarily serves as a preparatory step. It doesn't directly
address building understanding or relationships. Formal presentations may be one-way and
may limit engagement and understanding, as stakeholders have fewer opportunities to clarify
doubts or contribute input. This can also lead to a more passive relationship with the project.
While providing project documents is valuable, relying solely on individual review lacks the
direct interaction and discussion necessary for building shared understanding and strong
relationships. Stakeholders might have different interpretations or questions that wouldn't be
addressed in this format.
This question and rationale were developed in reference to:
https://www.pmi.org/learning/library/project-relationships-stakeholder-circle-8092
36. What leadership quality must a project manager have?
A. Honesty
B. Aggressiveness
C Powerful
D. Servant leader
Solution: A. Honesty
Honesty is a leadership quality that a project manager must possess. Project managers must
report the realities of the project and should be transparent. Aggression and power are too
forceful, and servant leadership is more for agile practices.
This question and rationale were developed in reference to:
https://www.projectmanagement.com/blog-post/67865/How-Will-Citizen-Development-
Impact-Leadership--Decision-Making-and-How-Projects-are-Run--Part-1 [Item
https://www.projectmanagement.com/blog-post/67865/How-Will-Citizen-Development-
Impact-Leadership--Decision-Making-and-How-Projects-are-Run--Part-1
How Will Citizen Development Impact Leadership, Decision-Making and How Projects are Run?
Part 1
37. In a hybrid project, the project manager receives a notification that, in response to a recent
court decision, a new project requirement must be incorporated into the project scope. What
should the project manager do?
A. Accept the new requirement as part of the project scope.
B. Exclude the new requirement from the project scope.
C. Submit a change request to the change control board (CCB).
D. Incorporate the new requirement after meeting with stakeholders.
Solution: C. Submit a change request to the change control board (CCB).
The CCB is responsible for reviewing, evaluating, approving, delaying, or rejecting changes to
the project, and for recording and communicating such decisions. In a hybrid project, it is
important to balance the need for agility and adaptability with the need for control and
predictability. Following a formalized change control process and submitting a change request
to the CCB will help to achieve this balance by providing a structured way to assess and approve
changes to the project scope. The CCB will evaluate the impact of the new requirement on both
the predictive and agile aspects of the project and make an informed decision.
The other options are incorrect because they lack a structured evaluation process and may lead
to uncontrolled scope changes.
It's crucial to avoid hasty decisions and assess the impact of the new requirement before
accepting or excluding it from the scope. In a hybrid project, maintaining agility and adaptability
is essential, and outright exclusion without evaluation can limit these qualities. Bypassing the
formal change control process can lead to uncontrolled scope expansion.
Incorporating the new requirement after meeting with stakeholders does not follow the formal
change management process. After the CCB reviews and approves the change request, the
project manager can then involve stakeholders to ensure alignment and understanding.
This question and rationale were developed in reference to:
| Project management : A Systems Approach to Planning, Scheduling, and Controlling (2009)
Harold Kerzner/John Wiley/10.1/ [Item]
PMBOK Guide Seventh Edition (2022) /// [2.6.2 DELIVERABLES]
38. An organization conducted an external audit in an Offshore Data Centre (ODC). The audit
team noticed that the project manager is using an old version of the risk register template. The
auditor raised a Non-Conformity (NC).
How could the project manager have avoided this gap?
A. It is not the project manager’s mistake since they are using the risk register template
provided by their organization
B. It is always better to conduct one round of internal audit before allowing an external audit to
avoid such a gap
C. It is suitable to conduct a risk management plan before preparing the risk register
D. It is appropriate to update the risk register with a new template and close the NC
Solution: C. It is suitable to conduct a risk management plan before preparing the risk register
The risk management plan may be developed as part of the project kick-off meeting, or a
specific planning meeting may be held. Attendees may include the project manager, project
team members, key stakeholders, or other team members who are responsible for the risk
management process for the project. Others outside the organization may also be invited, as
needed, including customers, sellers, and regulators. A skilled facilitator can help participants
remain focused on the task, agree on key aspects of the risk approach, identify and overcome
sources of bias, and resolve any disagreements that may arise. Additionally, as part of the risk
plan, monitoring the risk process should be conducted periodically to ensure that the risk plan
and risk register are properly updated.
The other options are incorrect. It is not a matter of having the right template to secure
updated registers. Project managers should comply with the risk plan to have an updated risk
register, and reviewing it periodically is part of the monitoring risk process. Reactive actions do
not guarantee to avoid the problem again.
This question and rationale were developed in reference to:
PMBOK Guide Sixth Edition (2018) PMI/PMI/11 Project Risk Management/404 [Item The risk
management plan may be developed as part of the project kick-off meeting or a specific
planning meeting may be held. Attendees may include the project manager, selected project
team members, key stakeholders, or team members who are responsible to manage the risk
management process on the project. Others outside the organization may also be invited, as
needed, including customers, sellers, and regulators. A skilled facilitator can help participants
remain focused on the task, agree on key aspects of the risk approach, identify and overcome
sources of bias, and resolve any disagreements that may arise.]
PMBOK Guide Sixth Edition (2018) PMI/PMI/11 Project Risk Management/457-458
39. Tools, such as checklists, brainstorming, SWOT analysis, and assumptions analysis are used
for which of the following?
A. Risk sharing
B. Risk identification
C. Risk analysis
D. Risk enhancement
Solution: B. Risk identification
Tools like checklists, brainstorming, SWOT analysis, and assumption analysis, are used to
identify risks.
The other options are incorrect because they do not typically use the tools listed. Risk sharing is
a risk response strategy whereby the project team allocates ownership of an opportunity to a
third party who is best able to capture the benefit of that opportunity. Risk analysis involves the
activities related to defining the characteristics of a risk and the degree to which it can impact
objectives. Risk enhancement is a risk response strategy whereby the project team acts to
increase the probability of occurrence or impact of an opportunity.
This question and rationale were developed in reference to:
https://www.pmi.org/learning/library/project-risk-management-issues-management-7267
PMBOK Guide Sixth Edition (2018) PMI/PMI/11 Project Risk Management/414-415 (11.2.2
Identify risks: tools and Techniques)
40. A project manager works at a private aerospace company. Risk management has become
one of the top priorities of the company.
Which is the first step the project manager needs to take while doing risk management?
A. Risk analysis
B. Risk acceptance
C. Risk identification
D. Risk register
Solution: C. Risk identification
The correct answer is risk identification.
Once the risk management scope and objectives are agreed upon, the process of identifying
risks begins, with care taken to distinguish genuine risks from nonrisks, such as concerns and
issues. It is unlikely that all risks are, or even can be, identified at the outset. Over time, the
level of risk exposure may change as a result of the decisions and actions taken previously and
of externally imposed changes
Risk analysis happens after risk identification.
Risk acceptance is a risk response strategy whereby the project team decides to acknowledge
the risk and not take any action unless the risk occurs.
The risk register is an output of the risk identification process that captures the details of
identified individual project risks.
This question and rationale were developed in reference to:
https://www.projectmanagement.com/articles/421532/Effectively-Managing-Risks-During-
Project-Execution [Item Effectively Managing Risks During Project Execution
Saleh Wadei - December 4, 2017]
The Standard for Risk Management in Portfolios, Programs, and Projects (2019)
41. A project manager is working on a critical project for which historical data is unavailable.
This may put the project at risk.
What should the project manager do?
A. Plan project execution based on stakeholders' risk appetites.
B. Develop the project management plan to identify and prioritize opportunities and threats.
C. Inform the sponsor of the risk involved, then continue with project delivery.
D. Develop a risk management plan and inform key stakeholders about the risks involved.
Solution: D. Develop a risk management plan and inform key stakeholders about the risks
involved
The correct answer is to develop a risk management plan and inform key stakeholders about
the risks involved. Historical records and data from past projects help to identify common risks
and prevent repeating mistakes. Historical data is used to identify systemic risks and automate
their treatment. Without historical data, the project manager must start from scratch to create
a detailed risk management plan. It is integral to the success of any project that stakeholders
are kept engaged and aware. This is especially important given the lack of historical data.
A risk management plan is a component of the project management plan that describes how
risk management activities will be structured and performed. A risk management plan should
align with the organization's risk tolerance. Planning the project based on stakeholders' risk
attitudes is not effective on its own. Not all stakeholders will have the same risk appetite, risk
management also needs to be aligned with the overall organization. Opportunities and threats
are identified as part of the risk management plan, which is a component of the overall project
management plan. Simply informing the sponsor will not provide a mitigation strategy should
the risk occur.
This question and rationale were developed in reference to:
PMBOK Guide Sixth Edition (2018) PMI/PMI/11/403 [Item]
| Project management : A Systems Approach to Planning, Scheduling, and Controlling (2013)
Harold Kerzner/John Wiley/17/ [Item]
42. A project manager is managing the build of an off-shore data center. In addition, the project
manager is the custodian of the assets procured by the project. The project manager is
informed by a team member that an asset was lost, and the team member is ready to pay for
the missing asset.
How should the project manager handle this situation?
A. Request your team member to purchase, replace and update the asset register with a new
asset tag
B. Inform the customer, purchase the new asset, and update the asset and risk register
C. Inform management, replace the asset with the help of an insurance claim, and update the
asset register
D. Inform customer and management immediately, launch an investigation, update the risk
register and take action based on outcome
Solution: D. Inform the customer and management immediately, launch an investigation,
update the risk register, and take action based on the outcome
The correct answer is to inform the customer and management immediately, launch an
investigation, update the risk register, and take action based on the outcome.
Risk management is everyone's job. Without an investigation, it may be difficult to accurately
identify where the failures were, who is responsible, how to respond, and how to prevent this
from happening again. Choosing not to inform the customer and management is inappropriate
because all risk owners and risk action owners must be briefed on any changes that may affect
their responsibilities. Updating the risk register is imperative as this is an incident that may be
repeated and may trigger corrective actions to avoid a repeat. There are a lot of unknowns here
that rule out the other options. Risk responses should be appropriate for the significance of the
risk, cost-effective in meeting the challenge, realistic within the project context, agreed upon by
all parties involved, and owned by a responsible person. Projects that are performed under
contract may have contract terms that specify how these types of situations should be handled.
Requesting that the team member purchase the replacement is incorrect because there should
be an investigation before making that decision. Further, the project manager is the custodian
of the assets, so the team member may not be considered financially responsible at all. We do
not know if there is an insurance policy associated with the asset or the extent of coverage, so
that is not necessarily an option.
This question and rationale were developed in reference to:
PMBOK Guide Sixth Edition (2018) PMI/PMI/11 PROJECT RISK MANAGEMENT/439
The Standard for Risk Management in Portfolios, Programs, and Projects (2019)
43. Several team members are discussing the project manager's management techniques and
the team cannot agree if the focus is on management or leadership. What is the valid option?
A. Acts as a manager as it focuses on control, efficiency and effectiveness
B. Act as Leader as you join People, Direction and Speed
C. Act as a manager as you enter Command and Doing the right things
D. None of the above
Solution: D. None of the above
This question and rationale were developed in reference to:
PMI-ACP Exam Prep, 2nd Ed edition (Oct, 15) Mike Griffiths/RMC/1/66 [Item]
44. You are managing a project with significant regulatory aspects. A vendor responsible for a
core module is undergoing an organization-wide transformation and it is unlikely that the
deliverable will be ready on time. Your risk mitigation plan is to use paper forms until the
module is implemented but this strategy is likely to slow down the frontline interactions.
What kind of risk could you introduce to the project?
A. Secondary risk
B. Residual risk
C. Primary risk
D. Compliance risk
Solution: A. Secondary risk
A secondary risk can be defined as a risk created by the response to another risk. In other
words, the secondary risk is a consequence of dealing with the original risk.
This question and rationale were developed in reference to:
https://www.pmi.org/learning/library/overall-project-risk-assessment-models-1386 [Item]
45. A project manager is responsible for a new agile project that is global in nature. After a few
sprints have been completed, the quality assurance team, based in a different country from the
PM, has generated a long defect list. The PM is certain that these issues are not related to
project quality, but are from language misinterpretation.
What should the project manager do to address these issues
A. Request additional budget to hire a local quality assurance team.
B. When the project is nearing completion, merge the development team with the quality
assurance team to test the deliverables.
C. Meet with the quality assurance team to criticize the list they provided.
D. While performing sessions with the full project team, review the project requirements and
their descriptions.
Solution: D. While performing sessions with the full project team, review the project
requirements and their descriptions
The best response is to review project requirements with the entire team at a meeting, so that
a common understanding can be reached, since the project manager believes that language
misinterpretations are the root of the problem.
Criticizing the quality team is inappropriate, as there is no indication that they are performing
incompetently. Since this is a global project, hiring a local team (which presumably speaks the
project manager's language) will not address the project needs to have diverse representation,
and would be unfair to the existing quality team.
Merging the quality and development teams near the end of the project is unwise, as it may
lose opportunities to address problems sooner. In addition, in many organizations, quality must
be assessed by a department independent of other functions.
This question and rationale were developed in reference to:
Information Technology Project Management, 7th Ed. (2015) Kathy Schwalbe/Course
Technology/2/68 [Item]
| O'Reilly Platform (No Date) //3/ [Item https://learning.oreilly.com/library/view/managing-
virtual-teams/9781631574061/Chapter_03.xhtml]
46. 33. A real estate organization conducts a monthly review of its strategic project portfolio.

Project A Project B

Start date March 2016 May 2017

Expected completion date April 2019 July 2020


Project manager PM-A PM-B

During a review in March 2018, it was found that Project B is experiencing problems that had
previously not been identified in its risk management process. With Project A starting
approximately one year prior, what information should be requested from Project A so that
Project B may take preventive and/or corrective actions?
A.Work performance reports
B.Lessons learned repository
C.Issue log
D.Activity list

Solution: B. Lessons learned repository


This portfolio contains a set of projects that are structured to achieve the organization's
strategic objectives. Since both projects are within this portfolio, the projects have similar
external factors. Thus the lessons learned from Project A may benefit Project B. The lessons
learned repository contains what went well, and what did not. All projects should update their
lessons learned registry throughout the lifecycle of the project so that other projects may
benefit.
The issue log will not list the challenges that Project B is facing since they are different projects.
The activity list for the two projects is different and will not contain what went well or not. The
work performance reports are high-level reports that will only provide variance analysis, earned
value data, and forecasting information.
This question and rationale were developed in reference to:
PMBOK Guide Sixth Edition (2018) PMI/PMI/4.4.3.1/104 [Item]
| The AMA Handbook of Project Management, Third Edition (2011) Paul C. Dinsmore and
Jeannette Cabanis-Brewin/AMACOM/Increase Opportunity for the success/4314 [Item]
47. A key stakeholder has submitted several requests for new functions for a product under
development. The project manager just completed the scope definition for the product's next
version release and communicated the scope statement to all stakeholders.
What should the project manager do next?
A. Revise the communications management plan to define the number of new feature and
function requests that can be submitted.
B. Review the acceptance criteria for the product deliverables as defined in the scope
statement.
C. Meet with stakeholders to emphasize their alignment with product objectives and the goals
of the project.
D. Review the scope management plan to determine how new functionality requests are to be
accommodated.
Solution: D. Review the scope management plan to determine how new functionality requests
are to be accommodated
The scope management plan documents how the project and product scope will be defined,
validated, and controlled. The key benefit of this document is that it provides guidance and
direction on how scope will be managed throughout the project.
The other choices are incorrect, as they have nothing to do with handling NEW changes to
project scope.
This question and rationale were developed in reference to:
Information Technology Project Management, 7th Ed. (2015) Kathy Schwalbe/Course
Technology/5/188-191 [Item]
| PMBOK Guide Sixth Edition (2017) //X3.3.3 EXECUTING PROCESS GROUP/ [Item]
| PMBOK Guide Sixth Edition (2018) PMI/PMI/5.3/147 [Item]
48. A project manager has started a new project with a 24 month timeline. Soon after the
kickoff, the sponsor informs the project manager that the timeline needs to be shortened to 6
months.
What should the project manager do in this scenario?
A. Focus on developing a minimum viable product (MVP) by referencing the product backlog for
items with the highest prioritization that can be completed in the new timeline.
B.Refer to the product backlog to identify the items that can be completed in the shortest
amount of time and add them to the project to complete in the new timeline.
C. Work with the sponsor to increase the budget to hire vendors to develop the deliverables in
the compressed timeline.
D. Implement fast tracking and record this timeline change in the risk register.
Solution: A. Focus on developing a minimum viable product (MVP) by referencing the product
backlog for items with the highest prioritization that can be completed in the new timeline
The best approach to meet such an aggressive timeline is to re-evaluate the scope based on
stakeholder value, and focus on the highest priority features that can be delivered within the
allotted time, i.e., the concept of Minimum Viable Product (MVP).
Simply identifying the items in the product backlog that take the least time to complete is
incorrect, as it does not consider stakeholder value, and could result in a product with lots of
low-value features.
The choices for increasing the budget to hire more resources (crashing) and fast tracking
(parallel pathing tasks normally done in series) are incorrect. Although they represent common
ways to compress a timeline and retain the full scope, it is questionable whether a project
schedule could be shortened by 75% with these techniques. It is more realistic to concentrate
on a reduced scope (MVP) that delivers the highest priority features.
This question and rationale were developed in reference to:
O'Reilly Platform (No Date) //Chapter 2/ [Item Agile Project Management: Creating Innovative
Products, Second Edition
FINAL THOUGHTS
Traditional project management methods focus on adhering to plans for scope, schedule, and
cost. But this formula often causes teams to deliver lower value. Plans rapidly go out of date,
but business goals and objectives tend to remain—how to meet them changes. By focusing on
value, both current and future, teams can align themselves with the goals of the organization
much more effectively. Whether it’s explicit valuing of product capabilities (Chapter 8),
embracing technical excellence (Chapter 9), or changing measures of success (Chapter 13),
defining practices that help teams deliver on “value over constraints” will be a major theme of
this book (see Figure 2-1).]
| O'Reilly Platform (No Date) //Chapter 7 Minimum Viable Product (MVP)/ [Item Getting to
Market With Your MVP]
| PMBoK, Sixth edition (2021) PMI/PMI/6/ [Item Schedule Compression]
49. A project manager and team have worked together in a predictive project management
environment for five years. The organization has decided to implement agile project
management and trains the project manager on this approach. The team begins their next
project using agile, but after five sprints the team still is dependent on the project manager to
make decisions related to the project.
How can the project manager help the team?
A. Regularly provide the team members feedback on all tasks.
B. Inform the team that they can make their own decisions.
C. Appoint a different project team with agile project management experience.
D. Support the team and mentor them on how they can make their own decisions related to
the project.
Solution: D. Support the team and mentor them on how they can make their own decisions
related to the project
The correct choice would be for the project manager to support and mentor the team in
making its own decisions. While predictive project management relies on the project manager
to direct the work, agile emphasizes self-organizing teams. Since the team in this question had
been operating in a predictive environment, they need training in agile methods, which the
project manager should promote.
Simply informing the team that they can make their own decisions is inadequate; the team
needs more support in adopting agile practices.
Regularly providing team members feedback on all tasks would be reverting to predictive
practices, and does not promote an agile environment.
Appointing an experienced agile team instead of developing the current one is unfair to the
existing team, and in doing so, the PM would be shirking their responsibility to support the
team.
This question and rationale were developed in reference to:
Essential Scrum: A Practical Guide to the Most Popular Agile Process (No Date) Kenneth S.
Rubin//10/185 [Item]
| O'Reilly Platform (No Date) //5 Key 3: Begin Devoloping Teams to Replace the Hierarchy/10
[Item The 3 Keys to Empowerment
by John P. Carlos; Alan Randolph; Ken Blanchard
Published by Berrett-Koehler Publishers, 1999]
50. The director of security compliance is upset because the compliance team has not been
informed about the product design decisions taken by the agile team members over the past
five iterations.
What should the project manager do to start correcting the situation?
A. Invite a delegate from the security compliance team to attend the daily standup meetings.
B. Ask the security compliance director to assign a delegate from the compliance team to work
full time with the agile team.
C. Meet with the security compliance director to determine a strategy to fulfill the compliance
requirements.
D. Ask the product owner to increase the budget to add iterations for the re-work needed for
compliance.
Solution: C. Meet with the security compliance director to determine a strategy to fulfill the
compliance requirements
The project manager should interactively collaborate to understand and implement the key
stakeholder's needs throughout the project duration, rather than assume a specific method of
interaction.
The other answer choices may come after meeting with the security compliance director and
reviewing the compliance requirements. This will start correcting the situation.
This question and rationale were developed in reference to:
Essential Scrum: A Practical Guide to the Most Popular Agile Process (No Date) Kenneth S.
Rubin//13/234 [Item]
| O'Reilly Platform (No Date) //10.3/ [Item Agile Foundations - Principles, practices and
frameworks by Peter Measey]
51. A team member is working on a critical task for a project, and approaches the project
manager immediately before the start of a project meeting and informs the project manager
that the task was not completed on time.
What should the project manager do?
A. Dismiss the team member from the meeting and ask him/her to continue working on
the task.
B. Reallocate the task to another team member and emphasize the importance of finishing
it on time.
C. Send an e-mail to the whole team and ask them for support in completing this specific
task.
D. During the meeting, review all open tasks with the team and jointly decide on which
action to take.
Solution: D. During the meeting, review all open tasks with the team and jointly decide on
which action to take
The best choice would be to review all open tasks during the meeting and use the team's
collective wisdom in assessing options and deciding how to proceed. While the project manager
might lead the discussion, the team can come up with more options than the project manager
alone.
The other choices are not the best, as they focus on efforts of a single team member or
completion of a single task, while excluding other perhaps more creative solutions.
This question and rationale were developed in reference to:
Choose Your WoW! A Disciplined Agile Delivery Handbook for Opitimizing Your Way of Working
(January 2019) Scott W Ambler and Mark Lines//22/312 [Item]
52. An agile project team has implemented design changes over several iterations. The data
privacy officer is concerned because their team wasn't involved in these decisions.
How should the project manager address these concerns?
A. Ensure that a data privacy team member attends every future daily standup.
B. Request the data privacy officer to assign a data privacy team member to join the
project team.
C. Set up a meeting with the data privacy officer to identify and implement a process to
meet requirements.
D. Meet with the project owner to reverse the changes the project team made without
consulting the data privacy team.
Solution: C. Set up a meeting with the data privacy officer to identify and implement a process
to meet requirements.
Scheduling a meeting fosters open communication and allows for the collaborative
development of a process to integrate security considerations into future iterations.
The other answer choices are incorrect. While including data privacy in future standups is
beneficial, it might not address past decisions or establish a long-term communication strategy.
Adding a permanent team member might not be necessary. The best approach should be
determined collaboratively. Reversing changes disrupts the agile process and disregards past
efforts. The project manager should focus on moving forward with a clear data privacy process.
This question and rationale were developed in reference to:
Essential Scrum: A Practical Guide to the Most Popular Agile Process (No Date) Kenneth S.
Rubin//13/234 [Item]
PMBOK Guide (6th edition) Chapter 13: Agile Project Management: This chapter outlines
communication and collaboration strategies for Agile projects.
PMI Article - "Agile Security: Integrating Security into Your Agile Process":
https://www.pmi.org/learning/library/agile-project-management-mandate-changing-
requirements-7043
53. A project manager has been assigned to a big project. The project sponsor would like the
project manager to perform a stakeholder analysis.
Which points should the project manager consider while performing this analysis?
A. Interests, rights, ownership, knowledge, and contribution.
B. Interests, requirements, rights, knowledge, and impact.
C. Power, impact, influence, requirements, and expectations.
D. Project role, impact, rights, knowledge, and interests.
Solution: C. Power, impact, influence, requirements, and expectations.
While performing stakeholder analysis, the project manager should consider the power
dynamics, the impact stakeholders may have on the project, their level of influence, and their
requirements and expectations. This information helps the project manager understand the
stakeholders' perspectives, engage with them effectively, and manage their involvement in the
project.
The other answer choices are incorrect because they either miss or incorrectly state some key
aspects of stakeholder analysis.
This question and rationale were developed in reference to:
PMBOK Guide 7 pg 11 section 2.1.1.2 Understand and Analyze Stakeholders
https://www.pmi.org/learning/library/stakeholder-analysis-pivotal-practice-projects-8905
https://www.pmi.org/learning/library/stakeholder-management-task-project-success-7736
54. At the end of an iteration, a team member tells the project manager that a planned task is
unfinished because of an issue that appeared days ago but could not be resolved. Why should
the project manager discuss the issue during the retrospective to prevent this type of situation
in the future?
A. It is a timely opportunity to discuss issues and ideas for improvement to encourage
team-based problem solving.
B. It allows the project manager to address the issue during a demo, which leads to high
levels of buy-in from the team members.
C. The team member will have the opportunity to resolve the issue before the
retrospective.
D. The project manager can emphasize how to maximize the time available to resolve the
issue.
Solution: A. It is a timely opportunity to discuss issues and ideas for improvement to encourage
team-based problem solving
The retrospective allows issues with the execution approach to be identified and discussed in a
timely fashion along with ideas for improvements. Retrospectives are a primary tool to manage
project knowledge and develop the team through discussions of what is working well and team-
based problem solving.
A retrospective does not require presenting a demo, so that choice is not correct. In addition,
nothing in the question suggests that a demo would be helpful in preventing recurrence of the
issue.
The other choices speak to resolution of the issue at hand, but do not answer the question,
which asks how a retrospective might help in preventing future occurrences.
This question and rationale were developed in reference to:
PMBOK Guide Sixth Edition (2018) PMI/PMI/Apendix X3/X3.3.3 [Item]
| Project management the managerial process (2009) Erik W. Larson/McGraw-Hill/17/590/591
[Item]
55. A project manager is working with a new team on an agile project. The team is regularly
missing milestones and are unable to meet the pace of the sprints.
How should the project manager proceed?
A. Contact the sponsor to see if additional budget can be made available to incent the
team members financially to improve performance.
B. Reference earlier sprints completed by the team and modify future sprints to align with
the team’s velocity to enable success.
C. Reach out to stakeholders to identify individuals with more experience to replace the
slower project team members.
D. Train the team on how to use Kanban to improve their overall performance.
Solution: B. Reference earlier sprints completed by the team and modify future sprints to align
with the team’s velocity to enable success
"Referencing earlier sprints..." (Correct option) The best indicator of team capacity is the
historical velocity metric and the delivery plan should take that in account prior to any solid
commitment by the team.
"Contact the sponsor..." additional compensation will not necessarily lead to increased
productivity.
"Additional budget" will not determine the velocity of the team. Agile is all about team velocity
and not "individuals with more experience".
"Kanban" helps to limit work-in-progress for better efficiency.
This question and rationale were developed in reference to:
Coaching Agile Teams (00/00/0000) Lyssa Adkins//Coaching at the beginning/75 [Item]
| Succeeding with Agile (No Date) //15/ [Item]
56. After monitoring a product’s development after the first three releases, the product owner
wants to launch a product even though it lacks some features identified in the minimum viable
product (MVP). A stakeholder is not satisfied with this version and feels it needs to meet the
previously agreed-upon MVP definition.
How should the project manager proceed?
A. Review the positives and negatives of releasing the product in its current format and
bring this analysis to the project sponsor to decide.
B. Work with the stakeholder and the product owner to try and find alignment but support
the product owner as they are ultimately responsible for the product.
C. Schedule a meeting with all stakeholders and the product owner to review the project’s
Pareto Chart.
D. Empower the project team to decide how to proceed as the product’s development is
the project team’s responsibility.
Solution: B. Work with the stakeholder and the product owner to try and find alignment but
support the product owner as they are ultimately responsible for the product
"Work with the stakeholders..." (Correct option) On agile projects, teams are empowered to
make decisions. It is important to support the team through the process. In Agile, a product
owner is responsible for refining and prioritizing the items in the back log.
"Review the positives..." This option is not correct because the product owner must be involved
to resolve the issue and assist with the decision.
"Schedule a meeting..." A Pareto chart is a type of chart that contains both bars and a line
graph, where individual values are represented in descending order by bars, and the cumulative
total is represented by the line. Reviewing Pareto chart is not going to help us in this scenario
"Empower the project team..." The project team would not prioritize the items in the back log.
This question and rationale were developed in reference to:
O'Reilly Platform (No Date) //4.1 How Do You Stimulate an Agile Culture?/ [Item Agile
Leadership Toolkit: Learning to Thrive with Self-Managing Teams]
| O'Reilly Platform (No Date) //4/ [Item Agile Project Management: Creating Innovative
Products, Second Edition
The capability of self-organizing teams lies in collaboration—the interaction and cooperation of
two or more people to jointly produce a result. When two engineers scratch out a design on a
whiteboard, they are collaborating. When team members meet to brainstorm a design, they
are collaborating. When team leaders meet to decide whether a product is ready to ship, they
are collaborating. The result of any collaboration can be categorized as a tangible deliverable, a
decision, or shared knowledge.]
57. During the fourth iteration of an agile project, the customer requests a technical change to
the development of the project deliverable.
What should the project manager do in this situation?
A. Update the project scope for this change and inform the sponsor
B. Inform the project team of the request, have them decide how to proceed with
development, and have them coordinate the changes with the stakeholders.
C. Work with the stakeholders to decide how best to proceed and communicate the
process changes to the project team.
D. Calculate the impact of the customer’s request on the schedule performance index (SPI)
and communicate this to the sponsor.
Solution: B. Inform the project team of the request, have them decide how to proceed with
development, and have them coordinate the changes with the stakeholders
The best choice is for the project manager to notify the team of the request, and have them
proceed with the development.
The other choices involve first approaching the sponsor or stakeholders, which is not correct.
The customer requested a technical change, which is within the team's scope to decide how to
address. Since this is an agile project, the team will consider the change and develop options,
and involve the appropriate stakeholders.
This question and rationale were developed in reference to:
Effective Project Management: Traditional, Agile, Extreme, Hybrid (No Date) Robert K.
Wysocki//9/ [Item]
| O'Reilly Platform (No Date) /// [Item Team Empowerment: 20 Ways to Get There
by Glenn Parker, Published by HRD Press, 2010, Empowered to do what?]
| The Agile Practice Guide (No Date) PMI/PMI/2/9 [Item]
58. The stakeholders and project team working on an agile project meet with the project
manager at the end of an iteration. The project manager learns that the schedule is slipping,
although all work was planned to meet the high-level objectives outlined in the project scope.
What should the project manager have done to keep the project on track?
A. Worked with the team to understand their expectations of using the agile approach.
B. Empowered team members to self-organize specific tasks to meet the objectives.
C. Prioritized work that could have had an early finish to allow for early benefits
realization.
D. Selected and sequenced the work in order to meet the high-level objectives.
Solution: B. Empowered team members to self-organize specific tasks to meet the objectives.
For agile projects, rather than a project manager prioritizing, selecting, and sequencing work,
they should ensure that higher-level objectives are explained and the team members are
empowered to self-organize specific tasks as a group to best meet those objectives. This leads
to the creation of practical plans with high levels of buy-in from the team members.
The other answer choices are incorrect. The project manager does not prioritize work or select
and sequence the work. The agile team prioritizes work after the product manager has decided
on a Minimum Viable Product (MVP). Working with the team to understand expectations does
not address prioritizing work.
This question and rationale were developed in reference to:
Fundamentals of Technology PM, 2nd Ed (2nd) Colleen Garton, Erika McCulloch/Mc Press/1/22
[Item]
| PMBOK Guide Sixth Edition (2018) PMI/PMI/Appendix 3/670 [Item]
59. A project management organization needs to assess the performance of two project teams.
Both projects have the same scope of work, but Team 1 works virtually and has calculated 100
story points, while Team 2 works in the office and has calculated 80 story points.
Which team has the better performance?
A. Team 1 because they work virtually.
B. Team 2 because they work in the office.
C. Team 1 with 100 story points performed better than Team 2 with 80 story points.
D. Team 2 with 80 story points performed better than Team 1 with 100 story points.
Solution: C. Team 1 with 100 story points performed better than Team 2 with 80 story points.

In Agile project management, story points are a measure of effort and complexity for the work
to be completed. A higher number of story points indicates a larger amount of work. Therefore,
Team 1, which has completed 100 story points, has accomplished more work than Team 2,
which has completed 80 story points. The number of story points completed is a key
performance indicator in Agile projects, and higher values generally indicate better
performance.
The other answer choices are incorrect. The opposite scenario suggests that Team 2 performed
better despite having fewer story points. This challenges the assumption that higher story
points indicate better performance. Team location does not necessarily factor in performance.
In this case, the virtual team performed better than the in-office team.
This question and rationale were developed in reference to:
O'Reilly Platform (No Date) //14.6.1.7/ [Item Book: Agile foundations]
| O'Reilly Platform (No Date) //2/ [Item Agile Metrics in Action: How to measure and improve
team performance
by Christopher W. H. Davis
Published by Manning Publications, 2015
https://learning.oreilly.com/library/view/agile-metrics-
in/9781617292484/kindle_split_020.html]

60. A project team has an after-hours get-together. One of the team members who was
recently hired and new to the team did not receive an invitation to the get-together. At the
next few standups, the new employee is visibly upset and does not participate much during the
sessions.
How should the project manager address this situation?
A. Schedule another after-hours get-together and invite the new team member.
B. Meet with the new team member to talk about their concerns.
C. During the next standup, ask the new team member to participate more.
D. Invite the new team member to meet after-hours to discuss the project.
Solution: B. Meet with the new team member to talk about their concerns

"Meet with the..." (Correct option) The individual may be affected by the situation, however,
the distraction could be something entirely unrelated. While the Servant Leader should
empower the team, this is a sensitive issue which is best handled at an individual level.
The other options do not address the question as there is a lack of investigation into the root
cause.
This question and rationale were developed in reference to:
Choose Your WoW! A Disciplined Agile Delivery Handbook for Opitimizing Your Way of Working
(January 2019) Scott W Ambler and Mark Lines//7/117 [Item]
| O'Reilly Platform (No Date) //8/ [Item Agile Foundations]
| The Agile Practice Guide (No Date) PMI/PMI//50 [Item "The servant leader together with the
team may decide to address other behaviors."]

61. A project manager is leading Project A, which is a project to update a customer’s database.
The project manager is monitoring international business news and reads of pending legislation
that would require companies to maintain a strict level of protection regarding customer
personal data. This new level of protection is stricter than what Project A currently achieves.
What should the project manager do in this scenario?
A. Meet with the customer to immediately update the level of customer data protection in
the project contract.
B. Bring this information to your organization’s legal department requesting that they
renegotiate the customer’s contract.
C. Note this information in the project risk register and work with the customer and the
sponsor to develop a mitigation plan.
D. Work with the organization’s legal department to update the contract to include
protections against the pending legislation.
Solution: C. Note this information in the project risk register and work with the customer and
the sponsor to develop a mitigation plan

The risk register should be regularly reviewed and as such, including the risk in the risk register
with a mitigation plan will ensure that this risk is regularly monitored and mitigated. There is no
need to update the contract immediately since the legal requirements are not yet certain.
Parking this risk till the next contract negotiation or waiting until the legal requirements are
certain might be too late to act. The risk can potentially turn out to be a non issue.
This question and rationale were developed in reference to:
Choose Your WoW! A Disciplined Agile Delivery Handbook for Opitimizing Your Way of Working
(January 2019) Scott W Ambler and Mark Lines//Section 2.9/133 [Item]
| PMBOK Guide Sixth Edition (2018) PMI/PMI/11/417 [Item]

62. A project manager is overseeing a project with an extremely tight schedule. During a
meeting, the project stakeholders express concerns about the project's ability to meet critical
business demands.
What should the project manager do in response to these concerns?
A. Work with the sponsor to revise the project scope to address stakeholder concerns.
B. Meet with the project team and stakeholders to identify the minimum viable product
(MVP).
C. Assure the stakeholders that the project is on schedule and will meet all business
demands.
D. Update the schedule and the communication plans to ensure the stakeholders are
updated.
Solution: B. Meet with the project team and stakeholders to identify the minimum viable
product (MVP).

The project manager must address the stakeholders' concerns about the project's ability to
meet critical business demands. Defining the minimum viable product would give the
stakeholders an overview of the business value it will deliver within the timeline. By focusing on
delivering the MVP first, the project team can increase the chances of meeting critical business
requirements, even with a tight schedule. Meeting with the project team and stakeholders
promotes collaboration, aligns priorities, and ensures that the project remains on track to meet
key business objectives.
The other options are incorrect because they do not appropriately address the stakeholders'
concerns.
Working with the sponsor to revise the project scope without understanding the minimum
viable product would not resolve the issue. Before taking action, the project manager must
understand the full scope of the stakeholders' concerns and the potential impact of any
changes to the scope. It is also important to get input from the project team to ensure that any
changes to the scope are feasible.
Assuring the stakeholders that the project is on schedule and will meet all business demands
does not address their concerns and may lead to mistrust. It's important to provide reassurance
based on actions and strategies.
Updating the schedule and communication plans does not directly address the stakeholders'
concerns. Updating the schedule and communication plan may or may not be necessary, and
can only be determined once the MVP is identified.
This question and rationale were developed in reference to:
Essential Scrum: A Practical Guide to the Most Popular Agile Process (No Date) Kenneth S.
Rubin//17/Product Roadmap Definition [Item]
PMBOK Guide Seventh Edition (2022) /// [3.3 EFFECTIVELY ENGAGE WITH STAKEHOLDERS]
[3.3.1 LIFE CYCLE AND DEVELOPMENT APPROACH SELECTION]

63. A project approval committee is concerned about a project plan that will take three years to
complete. The committee is concerned that the deliverable may not meet industry needs and
may be less valuable than anticipated.
What can the project manager do to demonstrate the value of the deliverable to stakeholders?
A. Add an additional sprint to the timeline for potential modifications.
B. Have the project team build a prototype to share with stakeholders.
C. Record the committee's expressed concerns in the issue log.
D. Add the committee to the communication management plan.
Solution: B. Have the project team build a prototype to share with stakeholders.

Building a prototype is one of the best ways to demonstrate the value of a deliverable to
stakeholders. Prototypes provide a tangible representation of the final product, allowing
stakeholders to visualize how it will work and look, which helps them understand its
functionality and usability. By sharing the prototype with stakeholders, the project team can
gather early feedback and make necessary adjustments to align the deliverable with industry
needs and expectations. It also helps mitigate risks related to delivering a final product that
may not meet stakeholders' requirements.
The other options are incorrect because they do not directly address the committee's concerns.
Adding an additional sprint may allow the project team to make modifications to the
deliverable, but it does not guarantee that the deliverable will meet industry needs or be
valuable to stakeholders. Extending the timeline can increase project duration and costs, which
might not be acceptable to stakeholders or necessary to address the value concern.
Recording the committee's concerns in the issue log captures the concerns but doesn't actively
address them or provide a tangible demonstration of value.
Adding the committee to the communication management plan will ensure that they are kept
informed of the project's progress, but it doesn't inherently address the committee's specific
concerns about the deliverable's value.
This question and rationale were developed in reference to:
PMBOK Guide Seventh Edition (2022) /// [2.4.2 PLANNING VARIABLES – Page 53]

64. A project manager is assigned to join a several year project mid-way. The project was
stopped years ago due to organizational change and a local financial crisis. It has now been
restarted. 70% of the budget is already consumed. The performance indices are CPI= 0.42 and
SPI= 0.2.During a meeting with stakeholders, the project manager also finds there are major
scope changes that could lead to delay.
What is the best course of action to take as project manager?
A. Change the earned value management tracking technique.
B. Change the contract according to new changes.
C. Negotiate scope changes with stakeholders and keep it minimized as possible.
D. Close this project and request a new project from the project sponsor.
Solution: D. Close this project and request a new project from the project sponsor

"Close this project...(Correct option)"Since there have been issues in the past and major scope
changes are anticipated, its better to close this project and charter a new project.
"Change the earned value management..." Changing the performance measurement technique
will not affect the project performance and there is no issue with the adopted performance
measurement technique i.e.; Earned Value Management.
"Change the contract..." Change the contract is incorrect as the changes are yet to be evaluated
and finalized.
"Negotiate scope changes..." with stakeholders is incorrect as it will not rectify the issues that
have occurred in the past and may lead to stakeholders disengagement.
This question and rationale were developed in reference to:
Agile Practice Guide (2017) PMI/PMI// [Item]
| PMBOK Guide Sixth Edition (2018) PMI/PMI// [Item]

65. A project manager is working on a new project that is expected to take 12 months to
complete. Due to the condensed timeline, the team must ensure that the small group of
stakeholders is actively engaged throughout the project. What should the project manager
emphasize as the most important activities in effective stakeholder engagement? (Choose 2)
A. Building the stakeholder map and maintaining it as the project progresses.
B. Prioritizing stakeholders and revisiting assumptions annually.
C. Creating more hierarchical structures among stakeholders.
D. Engaging key stakeholders and building their commitment to the project.
Solution: A and D. Building the stakeholder map and maintaining it as the project progresses
and Engaging key stakeholders and building their commitment to the project.

Building a stakeholder map is important because it helps the project manager to identify all of
the stakeholders, understand their interests and needs, and determine how they can be
engaged throughout the project.
Engaging key stakeholders is important because they can have a significant impact on the
success or failure of the project. By engaging them early and often, the project manager can
build their support and ensure that they are committed to the project's success.
The other options are incorrect because they are not as important in this case.
Prioritizing stakeholders and revisiting assumptions annually is a good practice, but it is not as
critical for a small group of stakeholders with a condensed timeline. Further, the project is only
intended to take 12 months to complete, so revisiting assumptions annually is not necessary.
Creating more hierarchical structures among stakeholders can be helpful in some cases, but it is
not necessary in this case.
This question and rationale were developed in reference to:
https://www.pmi.org/learning/library/engaging-stakeholders-project-success-11199 [Item
https://www.pmi.org/learning/library/engaging-stakeholders-project-success-11199]
PMBOK Guide Seventh Edition (2022) /// [2.1.1 STAKEHOLDER ENGAGEMENT]

You might also like